Prodotti

Messaggioda karl » 20/09/2005, 15:45

Visto che LupoGrigio ha sollevato l'interessante
problema dei prodotti infiniti ,ve ne propongo uno
io (anche questo abbordabile):
Immagine
Ciao.
karl
 

Messaggioda Fury » 20/09/2005, 17:32

Non vorrei dire 'na cavolata, però secondo me viene 1:
il risultato della produttoria sarà una funzione frazionaria
con il numeratore e il denominatore entrambi di grado n(->+00),
quindi, avendo lo stesso grado, il limite per n che tende a +inf
è uguale al rapporto dei coefficienti di grado massimo, che è sempre 1.
credo [:)]
Fury
Junior Member
Junior Member
 
Messaggio: 36 di 149
Iscritto il: 05/09/2005, 11:08
Località: Italy

Messaggioda MaMo » 20/09/2005, 17:41

No Fury. Il risultato è 2/3. Però bisogna dimostrarlo .....
Avatar utente
MaMo
Advanced Member
Advanced Member
 
Messaggio: 408 di 2108
Iscritto il: 27/04/2003, 17:20
Località: Sassuolo (MO)

Messaggioda Fury » 20/09/2005, 17:44

si, è vero, ho appena visto graficamente con derive.... ma teoricamente, perchè viene così?
Fury
Junior Member
Junior Member
 
Messaggio: 37 di 149
Iscritto il: 05/09/2005, 11:08
Località: Italy

Messaggioda Woody » 20/09/2005, 17:46

Il prodotto converge (lo si può vedere trasformandolo in una serie di logaritmi). Però come fai a dire che fa 2/3?

Woody
Woody
Junior Member
Junior Member
 
Messaggio: 128 di 223
Iscritto il: 13/02/2005, 14:05
Località: Italy

Messaggioda Fury » 20/09/2005, 17:54

non riesco a capire! mi sembrava proprio che il mio ragionamento filasse...
sto provando a calcolare qualche prodotto parziale, ma niente, mi viene sempre
uguale a 1. 2/3 l'ho visto solo graficamente!
Fury
Junior Member
Junior Member
 
Messaggio: 38 di 149
Iscritto il: 05/09/2005, 11:08
Località: Italy

Messaggioda MaMo » 20/09/2005, 18:01

Piccolo indizio: io lo avrei chiamato "prodotto telescopico".
Avatar utente
MaMo
Advanced Member
Advanced Member
 
Messaggio: 410 di 2108
Iscritto il: 27/04/2003, 17:20
Località: Sassuolo (MO)

Messaggioda Fury » 20/09/2005, 18:28

ho scoperto come fare!!!!! ora se avete un attimo di pazienza posto un'immagine!
Fury
Junior Member
Junior Member
 
Messaggio: 39 di 149
Iscritto il: 05/09/2005, 11:08
Località: Italy

Messaggioda Fury » 20/09/2005, 18:39

falso allarme, ma può portare sulla giusta strada! provate a guardare!
Immagine
Fury
Junior Member
Junior Member
 
Messaggio: 40 di 149
Iscritto il: 05/09/2005, 11:08
Località: Italy

Messaggioda Thomas » 20/09/2005, 20:47

scusate se rubo questo problema, ma mi devo rilassare con un bel problemino (sono un pò teso per vari motivi ed inoltre nn ho nulla da fare). Questa è la mia soluzione...

Scompongo con le identità:

a^3-1=(a-1)(a^2+a+1)
a^3+1=(a+1)(a^2-a+1)

e riscrivo riarrangiando:

f(n)=[(2-1)(3-1)(4-1)...(n-1) / (2+1)(3+1)...(n+1)] *
[(2^2+1+2)(3^2+1+3)...(n^2+1+n) / (2^2+1-2)(3^2+1-3)...]...(n^2+1-n)

forse è meglio la forma compatta:

f(n)= Prod[i=2..n] (i-1)/(i+1) * Prod[i=2..n] (i^2+1+i)/(i^2+1-i)

Le due produttore sono due telescopice, la prima dà subito 2/[n(n+1)], mentre per la seconda si osserva che

(i+1)^2+1-(i+1)=i^2+i+1

e la seconda telescopica semplificando in virtà dell'identità sopra dà (se non ci si crede la si scriva per esteso con i numeri!)

(n^2+n+1)/3

quindi mettendo assieme

f(n)= 2/3 *(n^2+n+1)/(n^2+n)

con limite 2/3

ciao
Thomas
Advanced Member
Advanced Member
 
Messaggio: 280 di 2223
Iscritto il: 28/09/2002, 21:44


Torna a Algebra, logica, teoria dei numeri e matematica discreta

Chi c’è in linea

Visitano il forum: francicko e 1 ospite